Você está na página 1de 7

Problemas e Teoremas em Teoria dos Números

Alex Abreu e Samuel Feitosa


7 de março de 2008

Nosso objetivo será apresentar algumas idéias e teoremas que consideramos indispensáveis para seu treinamento. Assumiremos
alguns resultados básicos que o leitor encontrará facilmente na referências [Plı́nio] e [EUREKA2]. Antes de ler a solução de
algum problema tente resolvê-lo sozinho. Se você quer se tornar um bom ”resolvedor”, você deve gastar muito tempo pensando
nos problemas para aprender a ter suas próprias idéias. Estude as soluções até dominá-las por completo. Dominar uma solução
significa que você consegue aplicar com facilidade as idéias ali contidas em outros problemas.

1 Divisibilidade
Nosso primeiro teorema é bem ingênuo, porém muito útil.
Teorema 1. Se b 6= 0 e a | b então |a| ≤ |b|.
Problema 1. (Leningrado 1990) Sejam a e b números naturais tais que b2 + ba + 1 divide a2 + ab + 1. Prove que a = b.

Solução. É fácil ver que a ≥ b e b2 + ab + 1 > a − b. A igualdade b(a2 + ab + 1) − a(b2 + ba + 1) = b − a implica que a − b é
divisı́vel por b2 + ba + 1. Se a − b 6= 0 então b2 + ab + 1 ≤ a − b ⇒ b2 + ab + 1 ≤ a − b. . Mas isso é um absurdo, logo a − b = 0.
Problema 2. (IMO 1998) Determine todos os pares de inteiros positivos (x, y) tais que xy 2 + y + 7 divide x2 y + x + y.

Solução. A igualdade y(x2 y + x + y) − x(xy 2 + y + 7) = y 2 − 7x implica que y 2 − 7x é divisı́vel por xy 2 + y + 7. Se y 2 − 7x ≥ 0,


então como y 2 − 7x < xy 2 + y + 7, segue que y 2 − 7x = 0. Então (x, y) = (7t2 , 7t) para algum t ∈ N. É fácil checar que esses
pares são realmente soluções. Se y 2 −7x < 0, então 7x−y 2 > 0 é divisı́vel por xy 2 +y +7. Mas daı́, xy 2 +y +7 ≤ 7x−y 2 < 7x,
que produz y ≤ 2. Para y = 1, temos x + 8 | 7x − 1 ⇒ x + 8 | 7(x + 8) − (7x − 1) = 57. Então as únicas possibilidades são
x = 11 e x = 49 e os pares correspondentes são (11, 1), (49, 1) que obviamente são soluções. Para y = 2 temos 4x + 9 | 7x − 4
e consequentemente 7(4x + 9) − 4(7x − 4) = 79 é divisı́vel por 4x + 9. Nesse caso não obtemos nenhuma solução nova. Todas
as soluções (x, y): (7t2 , 7t)(t ∈ N), (11, 1) e (49, 1).
Problema 3. (Rússia 1963) Resolva em inteiros a equação:
xy xz yz
+ + =3
z y x

Problema 4. (Leningrado 1989) Seja A um número


√ natural maior que 1, e seja B um número natural que é um divisor de
A2 + 1. Prove que se B − A > 0, então B − A > A.
Quando o conjunto de soluções de um dado problema é infinito é intressante tentar buscar alguma relação entre essas soluções.
Frequentemente existe algum tipo de relação de recorrência(lembra do material 2?). O próximo problema é um clássico e
ilustra bem essa idéia.
a2 + b2
Problema 5. (IMO 1988) Sejam a e b dois inteiros positivos tais que ab + 1 divide a2 + b2 . Mostre que é um
ab + 1
quadrado perfeito.
a2 + b2
Solução. Seja = k ∈ N. Temos a2 − kab + b2 = k. Assuma que k não é o quadrado de um inteiro, implicando que
ab + 1
k ≥ 2. Observemos agora um par (a, b) que é solução de a2 − kab + b2 = k. Podemos assumir sem perda de generalidade
que a ≥ b. Para a = b temos k = (2 − k)a2 ≤ 0, então a > b. Observe a equãção do segundo grau x2 − kbx + b2 − k = 0,
que tem soluções a e a1 . Como a + a1 = kb, segue que a1 ∈ Z. Como a > kb ⇒ k > a + b2 > kb e a = kb ⇒ k = b2 , segue
b2 − k a2 − 1
que a < kb e daı́ b2 > k. Como aa1 = b2 − k > 0 e a > 0, segue que a1 ∈ N e a1 = < < a. Então o par
a a
(a1 , b) satisfz 0 < a1 < a e é solução da equação original. Veja que descobrimos uma maneira de gerar soluções a partir de
uma solução dada. Dentre todas os pares (a, b) que são soluções com a ≥ b, escolha aquele que tem a mı́nimo. Pelo processo
anterior podemos obter outro par com um a menor. Mas isso é um absurdo, logo k é um quadrado perfeito.
Problema 6. Seja m um inteiro positivo. Defina a sequência an por a0 = 0, a1 = m e an+1 = m2 an − an−1 . Prove que
a 2 + b2
a ≤ b é solução de = m2 se e somente se (a, b) = (an−1 , an ) para algum n.
ab + 1
Problema 7. (IMO 2003) Encontre todos os pares de inteiros positivos (m, n) tais que

m2
2mn2 − n3 + 1
é um inteiro positivo.
Problema 8. (IMO 2007) Sejam a e b inteiros positivos. Mostre que se 4ab − 1 divide (4a2 − 1)2 , então a = b.
n3 +1
Problema 9. (IMO 1994) Encontre todos os pares ordenados (m, n) onde m e n são inteiros positivos tais que mn−1 é um
inteiro.
Problema 10. (Seletiva Rioplatense 2001) Encontre todos os pares (m, n) de números naturais com m < n tais que m2 + 1
é um múltiplo de n e n2 + 1 é um múltiplo de m.
Solução. Afirmamos que todas as soluções são da forma (F2k−1 , F2k+1 ), k ≥ 0 (Fn é o n-ésimo termo da sequência de
2
Fibonacci). É fácil ver que F2k−1 F2k+3 = F2k+1 + 1 e portanto os pares anteriores são soluções. Seja P o conjunto das
soluções que não são da forma (F2k−1 , F2k+1 ). P contém um par (a, b) tal que a + b é mı́nimo. Suponhamos a < b(se
2
a = b ⇒ (a, b) = (1, 1) = (F−1 , F1 ) 6∈ P ). Como b | a2 + 1 ⇒ a2 + 1 = bb0 ⇒ b0 < a. É fácil ver que a | b0 + 1 e b0 | a2 + 1.
Logo (b0 , a) é uma solução com b0 + a < a + b ⇒ (b0 , a) 6∈ P ⇒ (b0 , a) = (F2k−1 , F2k+1 ) ⇒ F2k−1 b = b0 b = a2 + 1 = F2k+1
2

b = F2k+3 ⇒ (a, b) = (F2k+1 , F2k+3 ) 6∈ P . Logo P deve ser vazio.
Problema 11. Prove que para qualquer inteiro positivo m, existe um número infinito de pares de inteiros (x, y) satisfazendo
as condições:
1. x e y são primos entre si;
2. y divide x2 + m;
3. x divide y 2 + m.

2 MDC, MMC e o Teorema de Bézout.


O próximo resultado é extremamente útil. Recomendamos a leitura da referência [EUREKA12]
Teorema 2. (Bézout) Sejam a e b inteiros não nulos e d seu mdc. Então existem inteiros x e y tais que d = ax + by. Se a
e b são positivos podemos escolher x > 0 e y < 0, ou vice-versa.
Prova. Seja P = {ax + by|ax + by > 0 e x, y ∈ Z}. O conjunto P é não vazio pois 0 < a2 + b2 = a · a + b · b ∈ P . Seja
f o menor elemento de P . Claramente d = mdc(a, b)| f Como f, d > 0, para mostrarmos que d = f basta que f | d. Seja
a = qf + r, com q ∈ Z e 0 ≤ r < f . Assim 0 ≤ r = a(1 − qx) + b(−qy) ∈ Z. Como r < f ⇒ r = 0. Analogamente f | b.
Então f | mdc(a, b) = d. A outra parte é deixada para o leitor.
Problema 12. (O Problema Clássico) Se a e b são números naturais, mostre que mdc(2a − 1, 2b − 1) = 2mdc(a,b) − 1
Solução. Seja d = mdc(a, b). Pelo teorena de Bezout, existem inteiros positivos x, y tais que d = ax − by. Seja k =
mdc(2a − 1, 2b − 1). Vale a igualdade 2by (2d − 1) = (2ax − 1) − (2by − 1). Como k | 2a − 1 | 2ax − 1 , k | 2b − 1 | 2by − 1 e
mdc(2by , k) = 1, temos k | 2d − 1. Como 2d − 1 | 2a − 1 e 2d − 1 | 2b − 1 então 2d − 1 | k. Logo k = 2d − 1.
Problema 13. Mostre que para quaisquer inteiros positivos ı́mpares a e b

mdc(2a + 1, 2b + 1) = 2mdc(a,b) + 1

o problema anterior será muito útil na nossa próxima solução.


Problema 14. (Teste IMO 2003) Seja n um inteiro positivo, e sejam p1 , p2 , . . . , pn , primos distintos maiores que 3. Prove
que 2p1 p2 ...pn + 1 tem pelo menos 4n divisores.
Solução. Suponha que a afirmação seja verdadeira para n primos. Seja N 0 = pN = p(p1 p2 . . . pn ). Então 2N + 1 tem pelo
menos 4n divisores distintos e 2p +1, em virtude do problema anteiror, tem pelo menos um fator q que não divide 2N +1. Então
(2N + 1)(2p + 1)
M= tem pelo menos 2 · 4n divisores distintos. Como p > 4 ⇒ (N − 2)(p − 2) > 2 · 2 então 2(N + p) < N p.
3 0 0
Mas M < 2N +p então M 2 < 22(N +p) < 2N p + 1 = 2N ‘ + 1. Como M divide 2N + 1, 2N + 1 = M · M 0 (M 0 > M ). Para cada
0 0
divisor d de M , ambos d e d · M 0 são fatores de 2N + 1. Então 2N + 1 tem pelo menos 4n+1 divisores.
Problema 15. A sequência de Fibonacci é definida por: F1 = F2 = 1 e Fn+1 = Fn + Fn−1 se n ≥ 2. Prove que:
a) Fm+n = Fm−1 Fn + Fm Fn+1 ;
b) mdc(Fm , Fn ) = Fmdc(m,n) .
Problema 16. (URSS 1988) A sequência de inteiros an é dada por a0 = 0, an = P (an−1 ), onde P (x) é um polinômio cujos
coeficientes são inteiros positivos. Mostre que para quaisquer inteiros positivos m, k com máximo divisor comum d, o máximo
divisor comum de am e ak é ad .
Solução. Quando temos um polinômio com coeficientes inteiros é sempre bom lembrar que a − b | P (a) − P (b). Essa será
nossa principal ferramenta nesta solução.
1. am | amr . Provaremos por indução. Se am(r−1) ≡ 0 (mod am ) ⇒ am(r−1)+1 ≡ P (0) (mod am ) ⇒ am(r−1)+2 ≡ P (P (0))
(mod am ) ⇒ amr = am(r−1)+m ≡ P (P (. . . (P (0)) = am ≡ 0 (mod am ).
| {z }
m vezes

2. Se l | at e l | af ⇒ l | at−f (Supondo t > f ). (Deixaremos a prova dessa afirmação para o leitor).


Pelo teorema de Bezout, existem inteiros positivos x, y tais que mx − ky = d. Seja n = mdc(am , ak ). Como n | am | amx e
n | ak | aky , pelo item 2, n | amx−ky = ad . Mas ad | am e ad | ak , então ad | n. Portanto ad = n.
Teorema 3. Existe uma sequência arbitrariamente longa de números compostos consecutivos.
Problema 17. (TT 2001) Existe um bloco de 1000 inteiros consecutivos não contendo nenhum primo, a saber, 1001! +
2, 1001! + 3, . . . , 1001! + 1001. Existe um bloco de 1000 inteiros consecutivos contendo apenas um primo?
Solução. Considere um bloco qualquer de 1000 números consecutivos começando no número a, façamos o movimento de
”andar uma casa”com o bloco que consiste em considerar o bloco de 1000 números consecutivos começando em a + 1. O
número de primos em um bloco muda no máximo em uma unidade após um movimento. Como o primeiro bloco(1,2,. . . ,
1000) contém mais que 5 primos e o bloco 1001! + 2, 1001! + 3, . . . , 1001! + 1001 não contém nenhum, algum bloco entre eles
deverá ter exatamente 5 primos.
Observar o mdc sempre é uma idéia útil. Veja o próximo exemplo:
Problema 18. Se x e y são inteiros tais que 2xy divide x2 + y 2 − x, prove que x é um quadrado perfeito
Solução. Seja d = mdc(x, y), então x = da e y = db com mdc(a, b) = 1. Como d2 |d2 a2 + d2 b2 − da temos que d|a ⇒ a = dc.
Como x|y 2 obtemos d2 c|d2 b2 , ou seja, c | b2 . Veja que mdc(a, b) = 1 ⇒ mdc(c, b) = 1. Assim c = 1 e x = d2 c = d2 .
Problema 19. (Leningrado 1990) Um bolo é preparado para uma festa de aniversário em que estarão exatamente p ou q
pessoas (p e q são inteiros relativamente primos). Encontre o menor número de pedaços (não necessariamente iguais) em que
o bolo deve se dividido para que possamos partilhar igualmente o bolo em qualquer caso.
Problema 20. (Rússia 95) A sequência a1 , a2 , ... de naturais satisfaz mdc(ai , aj ) = mdc(i, j) para todo i 6= j Prove que
ai = i para todo i.
Teorema 4. mdc(a, b)mmc(a, b) = |ab|.
Problema 21. Encontre todas as funções f , definidas sobre o conjunto de todos os pares de inteiros positivos, satisfazendo
as seguintes propriedades:
f (x, x) = x, f (x, y) = f (y, x), (x + y)f (x, y) = yf (x, x + y).
Solução. Claramente mmc(x, x) = x e mmc(x, y) = mmc(y, x). Usando o teorema anteiror temos:
xy x(x + y)
(x + y)mmc(x, y) = (x + y) =y· = y · mmc(x, x + y)
mdc(x, y) mdc(x, x + y)
Temos então uma forte suspeita que f = mmc. Para provarmos isso vamos usar uma idéia parecida à demonstração do
teorema de Bézout. Seja S o conjunto de todos os pares de inteiros positivos (x, y) tais que f (x, y) 6= g(x, y) = mmc(x, y), e
seja (m, n) o par em S com a soma m + n minima. Se tivermos m = n teremos:
f (m, n) = f (m, m) = m = g(m, m) = g(m, n)
Então devemos ter m 6= n. Suponha sem perda de generalidade que m > n. Então
nf (m, n − m) = [m + (n − m)]f (m, n − m) = (n − m)f (m, m + (n − m))
n−m n−m
então f (m, n − m) = · f (m, n). Analogamente temos g(m, n − m) = · g(m, n). Daı́ f (m, n − m) 6= g(m, n − m),
n n
então (m, n − m) ∈ S. Isto provoca uma contradição pois m + n − m < m + n. Desse modo, S deve ser um conjunto vazio e
f (x, y) = mmc(x, y).
Problema 22.
a) Prove que para quaisquer inteiros positivos a e b a equação mmc(a, a + 5) = mmc(b, b + 5) implica a = b.
b) É possı́vel que mmc(a, b) = mmc(a + c, b + c) para a, b, c inteiros positivos?
3 Congruências, Ordens e Números Primos
Teorema 5. Se mdc(a, m) = 1 então existe um inteiro x tal que

ax ≡ 1 (mod m).

Tal x é único (mod m). Se mdc(a, m) > 1 então não existe tal x.
Definição 1. O número φ(m) é o número de inteiros positivos que são menores ou iguais a m e que são relativamente primos
com m.

Definição 2. Um conjunto {r1 , r2 , . . . , rk } é chamado sistema completo de restos módulo n se para todo x existe um único i
tal que x ≡ ri (mod n). Em particular k = n
Definição 3. Um conjunto {r1 , r2 , . . . , rk } é chamado sistema completo de ivertı́veis módulo n se para todo x com (x, n) = 1
existe um único i tal que x ≡ ri (mod n). Em particular k = φ(n)
Teorema 6. Seja mdc(a, m) = 1. Sejam r1 , r2 , . . . , rk um sistema completo de restos(invertı́veis) de restos módulo m. Então
ar1 , ar2 , . . . , ark é um sistema completo de restos(invetı́veis) módulo m.
Problema 23. (Banco IMO 1987)
a) Seja mdc(m, k) = 1. Prove que existem inteiros a1 , a2 , . . . , am e b1 , b2 , . . . , bk tais que todos os produtos ai bj (i =
1, 2, . . . , m; j = 1, 2, . . . , k) deixam restos diferentes ao serem divididos por mk

b) Se mdc(m, k) > 1 Prove que para quaisquer inteiros inteiros a1 , a2 , . . . , am e b1 , b2 , . . . , bk existem dois produtos ai bj e as bt
com (i, s) 6= (j, t) que deixam o mesmo resto ao serem divididos por mk.
Teorema 7. (Teorema de Fermat) Seja p um primo. Se p não divide a então

ap−1 ≡ 1 (mod p).

Para todo inteiro a, ap ≡ a (mod p)


Teorema 8. (Teorema de Euler) Se mdc(a, m) = 1 então

aφ(m) ≡ 1 (mod m)

Teorema 9. (Teorema de Wilson) Se p é primo, então

(p − 1)! ≡ −1 (mod p)

Teorema 10. Todo primo da forma 4n + 1 é a soma de dois quadrados.

Prova. Vamos precisar de um lema:

Lema: Se p é um primo da forma 4n + 1, existe um inteiro x tal que x2 ≡ −1 (mod p).

Como (p − i) ≡ −i (mod p) temos:


 
p−1 p+1 p+3 p−1 p−1
(p − 1)! ≡ 1 · 2 · . . . · · · · . . . · (p − 1) ≡ 1 · 2 · . . . · · − · . . . · (−2) · (−1) (mod p)
2 2 2 2 2
p−1
Pelo teorema de Wilson, (p − 1)! ≡ −1 (mod p). Como (−1) 2 = (−1)2n = 1 temos:
 2  2
p−1 p−1 p−1
−1 ≡ ( )! · (−1) 2 ≡ ( )! (mod p)
2 2

p−1
Seja x = ( )!.
2 √ √
Considere agora, o conjunto de todos os pares (m, n) com m, n ∈ {0, 1, . . . , b pc}. Como existem (1 + b pc)2 pares e
√ 2 √ 2
(1 + b pc) > ( p) = p , pelo menos duas diferenças da forma m − xn terão o mesmo resto na divisão por p. Digamos que

m1 − xn1 ≡ m2 − xn2 (mod p) com (m1 , n1 ) 6= (m2 , n2 ).



Defina a = |m1 − m2 | e b = |n1 − n2 |. Então, a, b ∈ {0, 1, . . . , b pc} e a ≡ ±xb (mod p). Como a2 ≡ x2 b2 ≡ −b2 (mod p),

temos que p | a + b . Mas 0 < a + b < p + p = 2p pois (m1 , n1 ) 6= (m2 , n2 ) e a, b ∈ {0, 1, . . . , b pc}. Logo p = a2 + b2 .
2 2 2 2
Por que só funciona com primos da forma 4n + 1? O próximo problema é uma boa ajuda para responder essa pergunta.
Problema 24. Mostre que se p é um primo da forma 4n + 3 então não existe x tal que x2 ≡ −1 (mod p)
p−1 p−1
Solução. Suponha que exista tal x, como p−1
2 é ı́mpar, x2 ≡ −1 (mod p) ⇒ (x2 ) 2 ≡ (−1) 2 (mod p) ⇒ xp−1 ≡ −1
(mod p). Mas isso contradiz o teorema de Fermat.
Problema 25. (Banco IMO 1984) Sejam m e n inteiros positivos. Mostre que 4mn − m − n nunca pode ser um quadrado
perfeito.
Solução. Como 4m − 1 deixa resto 3 por 4, algum de seus divisores primos deve deixar resto 3 por 4. Seja p primo, p ≡ 3
(mod 4) e p | 4m − 1. Suponha que x2 = 4mn − m − n = n(4m − 1) − m ≡ −m (mod p) ⇒ (2x)2 = 4x2 ≡ −4m ≡ −1
(mod p). Pelo problema anterior isso é um absurdo. Logo 4mn − m − n nunca pode ser um quadrado perfeito.
Problema 26. (Seletiva Rioplantense 2007) Ache todos os primos p, q tais que pp + q q + 1 seja divisı́vel por pq.
Solução. É fácil ver que a equação do problema implica que p | q q + 1 e q | pp + 1. Suponha sem perda de generalidade
que p < q(É evidente que p 6= q). Como mdc(p − 1, q) = 1, pelo teorema de Bezout, existem inteiros positivos x, y tais
que (p − 1)x = qy + 1. Pelo teorema de Fermat, temos 1 ≡ q (p−1)x ≡ q qy+1 ≡ (q q )y q ≡ ±q (mod p). Logo p | q ± 1. Se
mdc(q − 1, p) = 1, obtemos de modo análogo que q | p ± 1. Se mdc(q − 1, p) 6= 1 ⇒ p | q − 1. Então temos que analisar as
seguintes possibilidades:
1. p | q + 1 e q | p + 1
2. p | q − 1
Se p | q − 1 ⇒ q ≡ 1 (mod p) ⇒ −1 ≡ q q ≡ 1 (mod p) ⇒ p = 2 ⇒ q | 22 + 1 = 5 ⇒ q = 5. Temos a solução (p,q)=(2,5). Se
p | q + 1 e q | p + 1 ⇒ q − 1 ≤ p ≤ q + 1. Como p 6= q e p < q ⇒ p = q − 1 ⇒ q − 1 | q + 1 ⇒ q − 1 | 2 ⇒ p = q − 1 = 2 ⇒ q = 3.
Veja que (2,3) não é solução. Soluções (p, q) = (2, 5), (5, 2).
Problema 27. Mostre que se k > 1 então 2k−1 6≡ −1 (mod k)
Solução. Suponha que k | 2k−1 +1. Claramente k é ı́mpar. Seja k = pα α2 αr
1 ·p2 ·. . .·pr , onde p1 , p2 , . . . , pr são primos ı́mpares.
1

mi
Seja pi − 1 = 2 · ti onde os ti são ı́mpares(i = 1, 2, . . . r). Suponha sem perda de generalidade que m1 é o menor elemento de
{m1 , m2 , . . . , mr }. Como pα
i ≡ 1 (mod 2
i mi
≥ 2m1 ), segue que k = pα α2 αr
1 · p2 ·m. . . · pr ≡ 1 (mod 2
1 m1
) ⇒ k − 1 = 2m1 · u. Mas
2m1 u 2m1 u
2k−1
≡ −1 (mod k) ⇒ 2 ≡ −1 (mod k) ⇒ 2 ≡ −1 (mod p1 ) ⇒ 2 2 1 ut1
≡ (−1) = −1 (mod k) ⇒ 2(p1 −1)u ≡ −1
t1
p1 −1 u u
(mod p1 ). Mas isso contradiz o teorema de Fermat pois (2 ) ≡ 1 ≡ 1 (mod p1 ).
Problema 28. (Bulgária 1995) Encontre todos os números primos p e q tais que o número 2p + 2q é divisı́vel por pq.
(Dica: Use o problema anterior)
Problema 29. (Bulgária 1996) Ensontre todos os números primos p e q tais que

(5p − 2p )(5q − 2q )
pq
é um inteiro.
Problema 30. (Bulgária 1997) Encontre todos os números inteiros m, n ≥ 2 tais que
n n
1 + m3 + m2·3
n
é um inteiro
Solução. Claramente n é ı́mpar, mdc(m, n) = 1 e n > 2. Se n = 3, como mdc(m, n) = 1 devemos ter que m ≡ 1 (mod 3)
n n
pois caso contrário 1 + m3 + m2·3 ≡ 1 − 1 + 1 ≡ 1 (mod 3). É fácil ver que todo par (m, n) = (3k + 1, 3) é solução.
n+1
n n n m3 −1
Suponha agora n > 3 e seja k = ordn m. Se n > 3 ⇒ m3 6≡ 1 (mod n). Como 1 + m3 + m2·3 = segue que
m3n − 1
n+1
n | m3 −1 ⇒ k | 3n+1 . Logo k = 3n+1 . Pelo teorema de Euler, mφ(n) ≡ 1 (mod n) ⇒ k ≤ φ(n) ⇒ 3n+1 ≤ φ(n) ≤ n − 1,
uma contradição.
Problema 31. (Alemanha) Se n é um número natural tal que 4n + 2n + 1 é primo, prove que n é potência de 3.
Problema 32. (IMO 1971) Prove que a sequência 2n − 3(n > 1) contém uma subsequência de números primos entre si dois
a dois.
Teorema 11. (Bertrand-Chebychev) Se n > 3 existe pelo menos um número primo entre n e 2n − 2.
Problema 33. Mostre que qualquer n ≥ 7 pode ser escrito na forma p + m, com p e m naturais primos entre si, maiores
que 1 e p é primo.
lnm n
Solução. Aplicando o teorema anterior com l = > 3, o menor inteiro maior ou igual a , produzimos um primo p com
2 2
n
< p < n − 1. Tal primo não pode dividir n pois p < n < 2p. Então n = p + (n − p) satisfaz o enunciado.
2
Tente resolver o problema anterior sem usar o teorema 11.
1 1 1
Problema 34. Prove que se n > 1 e k são números naturais, então o número X = + + ... + não pode ser
n n+1 n+k
um inteiro.
Problema 35. Seja P o conjunto de todos os primos e seja M um subconjunto de P , tendo pelo menos três elementos e tal
que para qualquer subconjunto próprio A de M , todos os fatores primos do número
Y
p
p∈A

estão em M . Prove que P = M .


Definição 4. Sejam a, n inteiros primos entre si, definimos a ordem de a módulo n como sendo:

ordn a := min{d > 0 | ad ≡ 1 (mod n)}

Teorema 12. Se am ≡ 1 (mod n) então ordn a|m.


Teorema 13. ordn a|φ(n).
1
Problema 36. (Teste Cone Sul 2002) Encontre o perı́odo na representação decimal de .
32002
Problema 37. Prove que que se p é primo, então pp − 1 tem um fator primo congruente a 1 módulo p
pp − 1
Solução. Seja q um primo que divide . Como q | pp − 1 segue que ordq p | p. Se ordq p = 1 ⇒ q | pp − 1 ⇒ 0 ≡
p−1
pp−1 + pp−2 + . . . p + 1 ≡ 1 + 1 + . . . + 1 + 1 ≡ p (mod q). Mas isso é um absurdo pois p 6= q. Logo ordq p = p e pelo teorema
pp − 1
anterior obtemos p | φ(q) = q − 1. Daı́ todos os divisores primos de são congruentes a 1 módulo p.
p−1
Problema 38. (IMO 2003) Seja p um número primo. Demonstre que existe um número primo q tal que, para todo inteiro
n, o número np − p não é divisı́vel por q.
No problema anterior, é possı́vel mostrar que existem infinitos primos q que satisfazem o enunciado.
Problema 39. (Leningrado 1990) Prove que para todos os inteiros a > 1 e n , n|φ(an − 1).
Solução. Seja k = ordan −1 a. Como ak ≡ 1 (mod an − 1) ⇒ ak − 1 ≥ an − 1 ⇒ k ≥ n. Como an ≡ 1 (mod an − 1) ⇒ k = n.
Pelo teorema anterior n = k | φ(an − 1).
Problema 40. Dado um primo p, prove que existem infinitos naturais n tais que p divide 2n − n.
2k 2k−1
Solução. Se p = 2 a afirmação é evidente. Suponha p 6= 2. Pelo teorema de Fermat, 2(p−1) −(p−1)2k ≡ (2p−1 )(p−1) −1 ≡
1 − 1 ≡ (mod p). Então todo número da forma (p − 1)2k satisfaz o enunciado.
Problema 41. (Putnam 1972) Prove que não existe um inteiro n > 1 tal que n|2n − 1.
Solução. Suponha que o conjunto S = {n ∈ N|n > 1, n | (2n − 1)} é não vazio. Seja m o menor elemento de S. Claramente
m é ı́mpa e pelo teorema de Fermat temos m | 2φ(m) − 1. Seja d = mdc(m, φ(m)), pelo problema 12, como m | 2m − 1 e
m | 2φ(m) − 1 segue que m | (2d − 1). Como m > 1 e m | (2d − 1), segue que d > 1. Como d | m segue que d | 2d − 1. Como
d ≤ φ(m) < m o elemento d está em S e é menor que o menor elemento desse conjunto. Essa contradição mostra que S é
vazio.
Definição 5. a é chamado raiz primitiva módulo n se ordn a = φ(n).
Teorema 14. Seja p primo e d tal que d|p − 1, então o número de resı́duos com ordem d é φ(d). Em particular p tem φ(p − 1)
raı́zes primitivas.
Teorema 15. Seja g uma raı́z primitiva módulo n, então se x é primo com n existe um a tal que x ≡ g a (mod n).
Teorema 16. Seja g uma raı́z primitiva módulo n, então g a também é raı́z primitiva se e somente se mdc(a, φ(n)) = 1.
Teorema 17. n tem raiz primitiva se e somente se n = 2, 4, pα , 2pα onde p é primo.
Teorema 18. (Euler) Um inteiro a satisfazendo mdc(a, p) = 1 é o resı́duo de uma potência n-ésima nódulo p se e somente
se vale a relação:
p−1
a d ≡ 1 (mod p) com d = mdc(p − 1, n).
Prova. Suponha que o inteiro a com mdc(a, p) = 1 é o resı́duo de uma potência n-ésima nódulo p, então existe x com xn ≡ a
(mod p). Pelo teorema de Fermat,
p−1 p−1 n
a d ≡ (xn ) d ≡ (xp−1 ) d ≡ 1 (mod p).
Provemos agora que a condição também é suficiente. Seja g uma raiz primitiva módulo p. Como mdc(a, p) = 1, existe um
h, 0 ≤ h ≤ p − 2 e a ≡ g h (mod p). Substituindo na nossa relação,
h(p−1)
g d ≡1 (mod p).
h(p − 1)
Como g é uma raiz primitiva, p − 1 | ⇒ d | h ⇒ h = kd. Pelo teorema de Bezout, existem números naturais u, v
d
tais que d = nu − (p − 1)v ⇒ kd = knu − k(p − 1)v. Como a ≡ g h ≡ g k d (mod p), em virtude do teorema de Fermat,
h(p−1)
a ≡ ag d ≡ g kd+k(p−1)v ≡ g knu ≡ (g ku )n (mod p).

Basta fazer x = g ku .
Problema 42. Ache todos os inteiros positivos n tais que vale an+1 ≡ a (mod n) para todo a inteiro.
Solução. Primeiro vemos que n é livre de quadrados. Suponha que exista p primo com p2 |n, tome a := p. Logo pn+1 ≡ p
(mod p2 ) absurdo, pois pn+1 ≡ 0 (mod p2 ). Então an+1 ≡ a (mod n) para todo a se e somente se an+1 ≡ a (mod p) para
todo a, p com p primo satisfazendo p|n (Teorema chinês dos restos). Tomando a raiz primitiva (mod p) temos que an+1 ≡ a
(mod p) se e somente se p − 1|n. Logo basta achar todos os n livre de quadrados satizfazendo p − 1|n para todo primo p
que divide n. Escreva n = p1 p2 . . . pk com p1 < p2 < . . . < pk , pi primo. Logo mdc(p1 − 1, n) = 1 portanto temos que
p1 − 1 = 1 o que implica p1 = 2 (solução n = 2). Analogamente p2 − 1|n logo p2 − 1 = 1, 2, portanto p2 = 3 (solução
n = 6). Continuando, p3 − 1 = 1, 2, 3, 6, logo p3 = 7 (solução n = 42), p4 − 1 = 1, 2, 3, 6, 7, 14, 21, 42, logo p4 = 43 (solução
n = 42·43 = 1806), p5 −1 = 1, 2, 3, 6, 7, 14, 21, 42, 43, 86, 258, 301, 602, 903, 1806 mas em nenhum caso p5 seria primo, portanto
temos só as soluções anteriores.
Problema 43. (Eureka) Prove que se p é um primo da forma 4k + 3, então 2p + 1 também é primo se e somente se 2p + 1
divide 2p − 1.
n
Problema 44. Prove que todos os divisores dos números de Fermat 22 + 1, n > 1, são da forma 2n+2 k + 1.
Problema 45. (IMO 1990) Encontre todos os inteiros positivos n > 1 tais que
2n + 1
n2
é um inteiro.

Referências
[Sierpinski] Sierpinski, W., Elementary Theory of Numbers, Hafner, New York, 1964.
[IR] Ireland, K.: Rosen, M., A Classical Introduciton to Modern Number Theory, Springer-Verlag, 1982.
[Plı́nio] José Plı́nio de Oliveira, Introdução à Teoria dos Números, IMPA, 2000.
[Moreira-Saldanha] Moreira, C.G., Saldanha, N.C. (2003), Primos de Mersenne (e outros primos muito grandes), IMPA,
Publicações Matemáticas.
[OBM] Carlos Moreira, Edmilson Motta, Eduardo Tengan, Luiz Amâncio, Nicolau Saldanha e Paulo Rodrigues, Olı́mpiadas
Brasileiras de Matemática, 9a a 16a .
[EUREKA2] Eureka! 2: Divisibilidade, Congruências e Aritmética Módulo n, Carlos G. T. de A. Moreira.
[Olimpedia] http : //erdos.ime.usp.br
[Sato] http : //www.artof problemsolving.com/Resources/P apers/SatoN T.pdf
[EUREKA12] Eureka! 12: Como Fermat e Bézout Podem Salvar o Dia, Antonio Caminha M. Neto.

Você também pode gostar